ACCA论坛's Archiver

xping 发表于 2009-1-5 10:22

CMA Gleim

<p>&nbsp;[1] Gleim #: 1.1.1 -- Source: Publisher<br/>Below are the grocers’ demand schedules for Palm Valley Grapes. Assuming that John, Towny, and Dorothea are the only<br/>three customers of Palm Valley Grapes, which of the following sets of prices and output levels will be on the market<br/>demand curve?<br/>Price John Towny Dorothea<br/>of Grapes Qdx Qdx Qdx<br/>$6 0 1 0<br/>5 1 2 0<br/>4 2 4 0<br/>3 3 6 1<br/>2 4 8 2<br/>1 5 9 3<br/>A. ($6, 2); ($1, 17)<br/>B. ($5, 3); ($1, 17)<br/>C. ($4, 4); ($2, 12)<br/>D. ($4, 0); ($1, 9)<br/>Answer (A) is incorrect because demand would be 1 when the price is $6.<br/>Answer (B) is correct. This type of problem is solved by means of trial and error. Check each of the answer<br/>alternatives to determine whether both points represent a level of demand for a given price. Answer (B) is correct<br/>because total demand would be 3 at a price of $5, and total demand would be 17 (5 + 9 + 3) at a price of $1.<br/>Answer (C) is incorrect because demand would be 6, not 4, when the price is $4, and demand would be 14, not 12,<br/>when the price is $2.<br/>Answer (D) is incorrect because demand would be 6 when the price is $4 and 17 when the price is $1.<br/>[2] Gleim #: 1.1.2 -- Source: CMA 1293 1-10<br/>If a group of consumers decide to boycott a particular product, the expected result would be<br/>A. An increase in the product price to make up lost revenue.<br/>B. A decrease in the demand for the product.<br/>C. An increase in product supply because of increased availability.<br/>D. That demand for the product would become completely inelastic.<br/>Answer (A) is incorrect because reduced demand will drive the price downward.<br/>Answer (B) is correct. A consumer boycott will decrease the demand for a product (shift the demand curve to the<br/>left). This decrease in demand should lead to a lower price for the product assuming that supply is constant (the<br/>supply curve does not shift).<br/>Answer (C) is incorrect because supply remains unchanged in the short run.<br/>Answer (D) is incorrect because, if demand is inelastic, customers will continue buying the product regardless of<br/>the price; a boycott, however, means that consumers will stop buying the product.</p>

CMA2009 发表于 2009-1-24 02:59

 啥东东呢

JJ880417 发表于 2009-2-14 23:33

什么东西啊?

xiaolidong 发表于 2009-2-22 11:19

<p>啥东西??</p>

shijdt 发表于 2009-3-6 13:39

<p>谢谢</p><p></p>

Shion0415 发表于 2009-3-21 16:12

谢谢

carolineh 发表于 2009-4-12 08:25

请问这个是第几版的呢?

bingsir 发表于 2009-4-26 22:03

no contents?

shiv 发表于 2009-5-27 15:28

I have no gold coin

yuongfong 发表于 2009-6-11 13:14

great

celamicky 发表于 2009-6-25 21:19

<p>poor me</p>

ybeybe 发表于 2009-7-1 19:15

have a look

qy506 发表于 2009-10-16 23:02

<p>想要</p>

qy506 发表于 2009-10-16 23:05

怎样购买?

chopstick 发表于 2010-6-8 09:21

想要

chopstick 发表于 2010-6-8 09:21

想要

gengxf 发表于 2010-9-17 21:56

<p>没钱购买啊</p>

wsl8028 发表于 2011-1-2 00:03

很好啊

henrylv 发表于 2011-2-23 19:34

内容需要支付 <b><font color="red">3</font></b> 个金币方可查

leoncn30 发表于 2011-8-30 10:30

want to know

页: [1] 2

Powered by CFAspace Archiver   © 2004-2011 CFAspace.com